Download as docx, pdf, or txt
Download as docx, pdf, or txt
You are on page 1of 48

1.

A client comes to the hospital emergency department with complaints of severe right lower
abdominal pain characteristic of appendicitis. The client does not have any health insurance. The
nurse understands that legally the hospital has which obligation?
a. Refer the client to the nearest public hospital.
b. Provide uncompensated care in emergency situations.
c. Have a health care provider see the client before admission.
d. Respect the family’s requests to admit their family member to the hospital.

B) Federal law and many state laws require that hospitals must provide emergency care. The client can
be transferred only after the client has been medically screened and stabilized. The client must give
consent for the transfer, and there must be a facility that will accept the client. Options A, C, and D do
not fully address the legal requirements for emergency care.

The nurse is admitting a client with a diagnosis of posttraumatic stress disorder to the mental health
unit. The client is confused and disoriented. During the assessment, what is the nurse’s primary goal for
this client?

1. Explain the unit rules

2. Orient the client to the unit

3. Stabilize the client’s psychiatric needs

4. Accept the client and make the client feel safe

4) Note the strategic word primary, and focus on the subject—a client being admitted to the mental
health unit. Using Maslow’s Hierarchy of Needs theory, remember that when a physiological need does
not exist, safety needs take precedence. It is important to accept a client and make a confused and
disoriented client feel safe. Orientation and explaining the unit rules are part of any admission process
and are not specific to this client. Stabilizing psychiatric needs is a long-term goal.

Reference: Linda Anne Silvestri PhD RN FAAN, Angela Elizabeth Silvestri PhD - Saunders 2020-2021
Strategies for Test Success_ Passing Nursing School and the NCLEX Exam (2019, Saunders)

An adult is admitted to the emergency room with a laceration on the forearm and BP = 140/74, P = 110,
and R = 36. The client also complains of tingling around the mouth and in the fingers and toes. What
should the nurse expect to do initially?
1. Administer oxygen

2. Have the client breathe into a paper bag

3. Call the physician immediately

4. Encourage the client to do deep breathing exercises

2) The data suggest that the client is hyperventilating and in respiratory alkalosis. Having the client
breathe into a paper bag is an appropriate response. Oxygen would make the client worse. Deep
breathing exercises are not likely to be as effective as having the client breathe into a paper bag.

A preparedness plan for a community-wide communication network includes the local emergency
response system being activated. The individual person or agency who is notified first would be the

1. Local health officer commander.

2. FBI field office.

3. Department of Health and Human Services.

4. Centers for Disease Control and Prevention (CDC).

(1) Once the local emergency response system is activated, the local health officer commander (who has
been prechosen) is notified first, and he/she in turn notifies the FBI field office, HHS, and the CDC.

Healthcare workers’ exposure to radiation is measured by an instrument readout in units of rads (R). A
reading of 50 R tells the worker that if he stays in the exposed area 1 hour, he will receive a rad
exposure of

1. 50 rad.

2. 5 rad.

3. 0.75 rad.

4. 100 rad.
(1) One hour is equal to an exposure of 50 rad. Gray is also a unit of exposure; so the worker should
know that if he receives less than 0.75 Gy (when 1 Gray equals 100 rad), his exposure will be in the safe
range.

6.

7.

8.

9.

10.

11. The best rationale for informing clients about the smallpox vaccination process is to

1. Avoid a later lawsuit.

2. Meet government expectations.

3. Provide safety information.

4. Make clients comfortable in signing the permission form.

(3) The best rationale for providing information is the safety element, so the client will know how to
recognize side effects, care for the blister, and not contaminate others. Regulations state that clients
cannot sue following a vaccination. The government has specific regulations about vaccination being
voluntary, rather than a government expectation. A permission form must be signed and adequate
information would make the client more comfortable, but it is not a safety issue.

12. The most toxic of all chemical agents is/are

1. Cyanide

2. Vesicant
3. Pulmonary agents

4. Nerve agents

(4) The most toxic chemical agents are nerve agents, such as sarin, tabon, soman, GF, and VX. These
agents block acetylcholinesterase, which regulates activity in organs, glands, muscles, and the CNS. With
no ability to “turn off,” the body wears out and death occurs in a short time.

13.

14.

15. Place in order the steps for treating life-threatening conditions. _________

1. Control bleeding.

2. Treat shock.

3. Check breathing.

4. Use head-tilt method if airway is obstructed.

The answer is 3 4 1 2. After gathering equipment, check breathing. If the victim is not breathing or
airway is obstructed, use head-tilt/chin-lift method. Next, you would control bleeding, followed by
treating shock.

Reference: Sandra Fucci Smith_ Marianne P. Barba - Sandra Smith's review for NCLEX-RN-Jones &
Bartlett Learning (2016)
A nurse is using contact precautions for a client diagnosed with Clostridium difficile. While transferring
the client from the bed to the commode, the client has loose stool that falls on the floor. After
positioning the client on the commode, how should the nurse proceed to cleanse the floor?

1. Wipe up the stool with toilet paper and then clean the area with soap and water

2. Wipe up the stool with toilet paper and then clean the area with a 1:10 bleach-water solution

3. Call housekeeping personnel to come clean the floor now with the unit’s mop and bucket

4. Wipe up the stool and apply the alcohol-based hand wash to clean the area of stool

A health-care provider (HCP) writes orders to transfuse a unit of red blood cells (RBCs) to a client
admitted to an emergency department after a disaster. A nurse is completing the compatibility checks
between the client’s blood type, noted to be blood type B-positive, and the unit of blood is blood type
O-positive, which had been donated by the client’s spouse and obtained from the blood bank. Which
clinical judgment by the nurse preparing to administer the unit of blood is correct?

1. The unit of blood is of a different blood type but compatible with the client’s blood.

2. The unit of blood is of a different blood type and incompatible with the client’s blood.

3. The unit of blood is not the blood component that the HCP prescribed for the client.

4. The unit of blood will cause a hemolytic transfusion reaction and cannot be administered to the client.

ANSWER: 1 Type O blood is compatible for persons with type A, B, or AB blood because it does not have
an antigen on the erythrocyte (RBC). A person with type O blood is considered a universal donor. The Rh
positive indicates that the Rhesus antigen is present on the cell, whereas Rh negative indicates that the
Rh factor is not present. Rh-positive RBCs can only be administered to persons who are Rh positive,
whereas Rh-negative RBCs can be administered to persons who are Rh positive or Rh negative. Although
the client and spouse have different blood types, the RBCs are compatible. The unit of blood states that
it is red blood cells (RBCs). A hemolytic transfusion reaction will occur if there are ABO or Rh
incompatibilities.

Reference: Ohman, Kathleen - Davis Q and A for the NCLEX-RN Examination-F.A. Davis (2010)
A prison inmate is brought to the emergency department with complaints of chest pressure that
radiates up the jaw and down the left arm. The nurse overhears another employee speaking rudely to
the inmate client. Acting as the client’s advocate, the nurse tells the employee privately that all clients
are to be treated with equal respect and dignity. Which client right has the nurse protected?

A. The right to free speech.

B. The right to privacy.

C. The right to considerate and respectful care.

D. The right to confidentiality.

Answer: C. Every client has the right to considerate and respectful care. The right to free speech is not
violated in this example nor is it included in the Patient’s Bill of Rights. The client’s privacy and right to
confidentiality are not breached in this example.

A client is brought to the emergency department by ambulance in an altered state of consciousness. The
client does not answer questions appropriately, is alternately agitated and drowsy, and speaks loudly
and incoherently at intervals. The client’s significant other arrives in the emergency department and
asks the nurse for the results of the client’s urine drug screen. The client’s significant other states that
the client has used illegal drugs in the past. The significant other needs the information for personal
safety and the safety of the couple’s children. What is the best response by the nurse to the request for
the results of the urine drug screen?

A. The results of the urine drug screen are positive for illegal substances.

B. You are wise to ask for the urine drug screen results for your safety and the children’s safety. The
drug screen results are positive for the drugs you gave me information about.

C. Thank you for the information about the client’s history of illegal drug use. Because you live together
and have children together, I can give you those results.

D. Thank you for the information about the client’s history of illegal drug use. A complete health history
helps us give better care. The health information of every client is confidential and protected by law and
cannot be released without the client’s consent. Does the client have a Durable Power of Attorney for
Health Care listing you as the surrogate decision-maker?

Answer: D. The health information of every client is confidential, protected by law, and can only be
released with the client’s consent. Releasing the urine drug screen results, or any health information, to
the significant other without an executed Durable Power of Attorney for Health Care or the client’s
consent is a violation of client confidentiality and HIPAA regulations regarding the privacy of health
information.

A client is found unconscious in a public park and is brought to the emergency department by
ambulance. The client is treated for mild exposure and dehydration and regains consciousness. The
client tells the nurse that, “the drug rehabilitation facility threw me out because they found drug
paraphernalia in my room.” What action by the nurse would be most helpful to the client at this time?

A. The nurse should inform the client to be truthful about any recent drug use.

B. The nurse should place a referral to the social worker assigned to the emergency department. The
social worker will have information and resources to assist the client with appropriate placement and
shelter.

C. The nurse should place a referral to the social worker for a possible psychiatric evaluation. The client
could be a danger to self or others.

D. The nurse should call the client’s previous drug rehabilitation facility and inform them that the drug
paraphernalia was not the client’s and request that the client be accepted back into the facility.

Answer: B. The action by the nurse that will be most helpful at this time would be to place a referral to
the social worker, who will have information and resources to assist the client with appropriate
placement and shelter.

A friend contacts a nurse at work and states, “Our friend, _______, has just been in a car accident and is
in your emergency room. I am so worried. Can you check your hospital computer system and tell me the
extent of the injuries?” What is the most appropriate response by the nurse to the concerned friend?

A.I did not know about the accident, but I can’t look that information up for you. Every client’s health
information is confidential and is protected by law. Unless I am caring for that person, I have no right to
access their health information. I recommend that you contact the family for information.

B. I will look the information up, but do not tell anyone that I did. Health information is confidential and
is protected by law.

C. OK, let me see here—the CAT scan looks terrible! You had better come to the emergency department
as fast as you can!

D. I can’t look that up for you. I will get into trouble. Call the emergency department and see if they will
give you any information.

Answer: A. The nurse should inform the concerned friend that the client/friend’s health information is
confidential and protected by law and cannot be accessed by the nurse without a treatment necessity or
relayed to others without the client’s consent. This appears harsh to concerned friends and family
members calling to inquire about a loved one but the HIPAA regulations for the confidentiality of health
information were enacted to ensure the privacy of all health information. Medical personnel must be
kind but firm while explaining and enforcing these regulations.

A client presents to the emergency department with complaints of fever, cough, malaise, and rash
appearing as vesicles most prominently on the face, palms of the hands, and soles of the feet. The
nurse’s first reaction is to:

A. Send the client to the waiting room to wait for an available examination room.

B. Place the client into a negative pressure room implementing airborne precautions.

C. Notify the emergency department physician.

D. Place the client on contact precautions.

Answer: B. The client may have smallpox, which is very contagious. Smallpox can also be used in a
biological incident. The first thing the nurse should do is place the client into a negative pressure room
and implement airborne precautions, which require all persons to wear a fit tested N95 respirator.
Wearing a surgical mask is not sufficient in this case. Doing this first will protect others from potential
exposure to a very contagious disease. After the client is sequestered, the nurse should notify the
emergency department physician for further treatment instructions.

A chemical exposure event occurs during a football game. The hospital is notified and expects to receive
clients. Which statement is most important regarding the decontamination of clients who are
nonambulatory?

A. The nurse should don appropriate personal protective equipment (PPE) prior to contact with the
client.

B. Clothes should not be removed and the client should be transported to an emergency department to
receive lifesaving interventions.

C. Decontamination of the eyes is not required.

D. Hot water should be used to decontaminate the client.

Answer: A. PPE should be donned prior to contact with the client to prevent contamination of the health
care worker. All clothes, jewelry, and personal belongings should be removed and placed into
appropriate containers. Decontamination of the eyes is performed using a saline solution via nasal
cannula or Morgan lens. Hot water is unnecessary unless the client is hypothermic during
decontamination procedures.
A 20-year-old college student who lives in a dormitory is admitted to the emergency department with
complaints of headache, nausea, vomiting, stiff neck, and a rash. The nurse should perform which action
based on the information given:

A. Wear a fit-tested N95 mask when caring for the client.

B. Implement droplet precautions when caring for the client.

C. Use airborne precautions and place the client into a negative pressure room.

D. Implement standard precautions when caring for the client.

Answer: B. Neisseria meningitidis is usually found in college students, jails, or anywhere there is close
contact among people. Headache, nausea, vomiting, stiff neck, and a rash are signs and symptoms of
meningitis. The nurse should implement droplet precautions by wearing a surgical mask and eye
protection during close contact with the client until a diagnosis is made. Placing the client in a negative
pressure room is not necessary. Standard precautions do not protect the nurse against airborne
transmission of meningitis.

A client presents to the emergency department with a cough. The nurse’s assessment reveals the client
is also experiencing chills, fever, night sweats, and hemoptysis. The nurse suspects the client may have
which illness?

A. Active tuberculosis (TB).

B. Bronchitis.

C. Upper respiratory infection.

D. Pneumonia.

Answer: A. Classic signs and symptoms of active TB infection include fever, chills, night sweats, and
hemoptysis. The nurse may see cough, fever, and chills in bronchitis, an upper respiratory infection, and
pneumonia, but would not see hemoptysis and drenching night sweats.

A client arrives in the emergency department after a motor vehicle accident. The client has sinus
tachycardia, is hypotensive, and has muffled heart sounds. There is no apparent sign of hemorrhage.
Which condition does the nurse suspect?

A. Cor pulmonale.

B. Pneumothorax.

C. Cardiac tamponade.

D. Pulmonary embolism.

Answer: C. Cardiac tamponade produces muffled heart sounds and signs and symptoms of shock
(hypotension and tachycardia). Cor pulmonale (right heart failure in the absence of left heart failure)
produces liver enlargement, abdominal pain, and jugular venous distension. Pneumothorax causes
diminished breath sounds, respiratory distress, and tracheal displacement. In clients with pulmonary
embolism, shortness of breath, apprehension, and hemoptysis are present.

A client complains of crushing chest pain 3 hours prior to arrival in the emergency department. Initial
vital signs show hypotension; a weak, thready pulse; cool, clammy skin; and confusion. Which
intervention should the nurse perform first?

A. Airway management.

B. Intravenous access.

C. Obtaining an EKG.

D. Preparing for intra-aortic balloon pump.

Answer: A. This client exhibits signs of cardiogenic shock, a complication of myocardial infarction.
Hypotension accompanied by clinical signs of increased peripheral resistance (weak, thready pulse and
cool, clammy skin) and inadequate organ perfusion (altered mental status and decreased urine output)
are found in this client. Airway management is always the most important intervention. Intravenous
access, obtaining an EKG, and possible preparation for an intra-aortic balloon pump are interventions
that occur after the initial intervention of airway management.

A client is using a leaf blower near an old campfire. Glass debris from a broken bottle fly everywhere.
The client comes to the emergency room complaining of a foreign body sensation in his right eye,
watery eyes, and photophobia. Which nursing action takes priority?

A. Evert eyelid and examine for foreign body.

B. Measure visual acuity.

C. Notify immediately for transfer.

D. Place an eye shield over eye.

Answer: D. If a foreign body is the result of explosion or blunt or sharp trauma, the eye should be
protected from further damage by placing an eye shield over the eye (or if a shield is not available, a
paper cup to prevent rubbing of the eye). After this is done, arrangements should be made to transport
the client for emergency care by an ophthalmologist. Everting of the eyelid, examining for foreign body,
and measuring visual acuity are not measures that should be performed immediately.

A client comes to the emergency department following a motor vehicle collision with questionable loss
of consciousness. To quickly rule out a serious injury, the nurse should first:
A. Assess airway patency, breathing, and circulation.

B. Assess level of consciousness.

C. Measure vital signs.

D. Stabilize neck and check for signs of neck injury.

Answer: A. The nurse should always assess airway patency, breathing, and circulation first. Assessing
level of consciousness, measuring vital signs, and stabilizing the neck, and checking for signs of neck
injury are all measures that are taken after the nurse initially assesses airway patency, breathing, and
circulation.

Reference: (Hurst reviews) Hurst, Marlene - NCLEX-RN review-McGraw-Hill Medical (2008)

At the scene of a train crash, the nurse triages the victims. Which clients should be coded for triage as
most urgent or the first priority? Refer to chart. Select all that apply.

1. Is dead
2. Has chest pain
3. Has a leg sprain
4. Has a chest wound
5. Has multiple fractures
6. Has full-thickness burns over 30% of the body

Answer: 2, 4, 6

Rationale: In a disaster situation, saving the greatest number of lives is the most important goal. During
a disaster the nurse would triage the victims to maximize the number of survivors and sort the treatable
from the untreatable victims. Prioritizing victims can be done in many ways, and many communities use
a color-coding system. First priority victims (most urgent and coded red) have life-threatening injuries
and are experiencing hypoxia or near hypoxia. Examples of injuries in this category are shock, chest
wounds, internal hemorrhage, head injuries producing loss of consciousness, partial- or fullthickness
burns over 20% of the body surface, and chest pain. Second priority victims (urgent and coded yellow)
have injuries with systemic effects but are not yet hypoxic or in shock and can withstand a 2-hour wait
without immediate risk (e.g., a victim with multiple fractures). Third priority victims (coded green) have
minimal injuries unaccompanied by systemic complications and can wait for more than 2 hours for
treatment without risk (leg sprain). Dying or dead victims have catastrophic injuries, and the dying
victims would not survive under the best of circumstances (coded black).

An emergency department nurse is a member of an all-hazards disaster preparedness planning group.


The group is developing a specific emergency response plan in the event that a client with smallpox
arrives in the emergency department. Which interventions should initially be included in the plan?
Select all that apply.
1. Isolate the client.

2. Don protective equipment immediately.

3. Notify infectious disease specialists, public health officials, and the police.

4. Lock down the emergency department and the entire hospital immediately.

5. Identify all client contacts, including transport services to the emergency department and clients in
the waiting room.

6. Administer smallpox vaccines to all hospital staff, client contacts, and clients sitting in the emergency
department waiting room immediately.

Answer: 1, 2, 3, 5

Rationale: An all-hazards disaster preparedness group is a multifaceted internal and external disaster
preparedness group that establishes action plans for every type of disaster or combination of disaster
events. In the event of emergency department exposure to a communicable disease such as smallpox,
the client would be isolated immediately and the staff would immediately don protective equipment.
The emergency department would be locked down immediately. Locking down the entire hospital may
not be necessary and infectious disease specialists and public health officials will determine whether it is
necessary to take this action. Infectious disease specialists, public health officials, and the police are
notified. All client contacts (name, addresses, telephone numbers), including transport services to the
emergency department and clients in the waiting room, would be identified so that the public health
department can follow through on notifying and treating these individuals appropriately. Although
getting the vaccine within 3 days after exposure will help prevent the disease or make it less severe, it is
unreasonable and unnecessary to administer smallpox vaccines to all hospital staff, client contacts, and
clients sitting in the emergency department waiting room.

The nurse working on a medical nursing unit during an external disaster is called to assist with care for
clients coming into the emergency department. Using principles of triage, the nurse should implement
immediate care for a client with which injury?

1. Fractured tibia

2. Penetrating abdominal injury

3. Bright red bleeding from a neck wound

4. Open massive head injury, resulting in deep coma

Answer: 3

Rationale: The client with bright red (arterial) bleeding from a neck wound is in “immediate” need of
treatment to save the client’s life. This client is classified as an emergent (life-threatening) client and
would wear a color tag of red from the triage process. A green or “minimal” (nonurgent) designation
would be given to the client with a fractured tibia, who requires intervention but who can provide self-
care if needed. The client with a penetrating abdominal injury would be tagged yellow and classified as
“urgent,” requiring intervention within 60 to 120 minutes. A designation of “expectant” would be
applied to the client with massive injuries and minimal chance of survival. This client would be color-
coded “black” in the triage process. The client who is color-coded “black” is given supportive care and
pain management but is given definitive treatment last.

The nurse working on an adult nursing unit is told to review the client census to determine which clients
could be discharged if there are a large number of admissions from a newly declared disaster. The nurse
determines that the clients with which medical situations would need to remain hospitalized? Select all
that apply.

1. Laparoscopic cholecystectomy

2. Fractured hip, pinned 5 days ago

3. Diabetes mellitus with blood glucose at 180 mg/dL (10.2 mmol/L)

4. Ongoing ventricular dysrhythmias while receiving procainamide

5. Newly delivered postpartal client with a blood pressure of 146/94 mm Hg and 2+proteinuria.

Answer: 4, 5

Rationale: The client with ongoing ventricular dysrhythmias requires ongoing medical evaluation and
treatment because of potentially lethal complications of the problem. The newly delivered postpartal
client is showing classic signs for mild preeclampsia. This condition would need to be reversed before
discharge. Each of the other problems listed may be managed at home with appropriate agency referrals
for home care services and support from the family at home.

The nurse is caring for a client who is a survivor of a disaster event. The client begins to display
behaviors not demonstrated before. Which manifestations should indicate to the nurse that the client
may be experiencing post-traumatic stress disorder (PTSD)? Select all that apply.

1. Irritability and sleep disturbances

2. Flashbacks or recollections of the disaster

3. Regression to an earlier developmental stage

4. A feeling of estrangement or detachment from others

5. Consistent discussion and rationalizing as to why the disaster occurred

6. Repression or the inability to remember an important aspect associated with the disaster

Answer: 1, 2, 4, 6

Rationale: PTSD is characterized by a sustained maladaptive response to a traumatic event. In this


condition, the client experiences recurrent and intrusive recollections of the event (flashbacks), has
recurrent dreams of the event, acts or feels as though the event were recurring, and experiences
psychological distress when internal or external cues resemble the event. The individual avoids stimuli
associated with the trauma or event (thoughts, feelings, conversations about the event, and persons or
places that evoke memories of the event). The individual also is unable to remember an important
aspect of the event (repression) and experiences somatic symptoms such as irritability and sleep
disturbances. Regression to an earlier developmental stage and consistent discussion and rationalizing
as to why the disaster occurred are not characteristics of PTSD.

A client reports having difficulty concentrating and outbursts of anger, as well as feeling “keyed up” all
the time. The client reveals that the behaviors began soon after witnessing the murder of a good friend.
The nurse should suspect which stressor before communicating with the client?

1. Social phobia

2. panic disorder

3. post-traumatic stress disorder (PTSD)

4. obsessive-compulsive disorder (OCD)

Answer: 3

Rationale: PTSD is a response to an event that would be markedly distressing to almost anyone.
Characteristic symptoms include a sustained level of anxiety, difficulty sleeping, irritability, difficulty
concentrating, and outbursts of anger. Panic disorders and social phobia are characterized by a specific
fear of an object or situation. OCD involves some repetitive thoughts or behaviors.

The nurse comes on duty at 8 a.m. After she receives reports and begins to organize care for her select
group of patients, the fire alarm begins to ring. Which action taken by the nurse indicates that the nurse
is aware of fire safety?

(1) Close all doors on the unit.

(2) Begin to move patients for evacuation.

(3) Leave the unit to get help.

(4) Instruct all visitors to leave the building.

The correct answer is Choice (1). Remember RACE when answering similar test questions: » Rescue
patient » Activate alarm » Contain fire (or Close doors on the unit) » Extinguish In the example question
the alarm already has been activated, so the nurse correctly moves to the next step in the process — she
closes all doors to prevent fire or smoke from spreading. Hospital administration decides when to move
patients for evacuation, leaving the unit qualifies as abandonment, and instructing visitors to leave may
complicate rescue efforts. Choice (3) of leaving the unit is not a part of RACE, even to go get help. It’s
not rescuing, containing, or extinguishing the fire. Much of what is expected of nurses in providing a safe
and effective environment requires not only knowledge of specific nursing functions but also knowledge
of other disciplines and how they contribute to the safe choices made in nursing care.
An emergency department nurse is assigned to triage patients arriving for treatment after an auto
accident. The nurse should assign the highest priority to which of the following patients?

(1) The patient with inward movement of an area of his chest when inhaling and outward movement on
exhalation

(2) The patient with swelling and bruising to his knee

(3) The patient complaining of muscle aches and a backache

(4) The patient with small lacerations on his head with significant bleeding

The correct answer is Choice (1). Inward movement of the chest during inhalation describes paradoxical
breathing, which indicate respiratory distress. All other options are not life threatening.

The nurse enters a patient’s room and finds that the wastebasket is on fire. The nurse immediately
assists the client out of the room. What is the next nursing action?

(1) Confine the fire by closing the room door.

(2) Activate the fire alarm.

(3) Call for help.

(4) Extinguish the fire.

The correct answer is Choice (2). Keywords for this question are wastebasket is on fire, immediately
assists the client out of the room, and next nursing action. The first priority in the event of a fire is to
rescue clients who are in immediate danger, but the nurse in this question has already done that. Use
RACE to remember that the next step is activating the alarm. Then you confine the fire by closing all
doors and lastly extinguish the fire.

A nurse enters a patient’s room and discovers that the chair is on fire. She rescues the patient, pulls the
alarm, closes the door, and obtains the fire extinguisher to put out the fire. The nurse pulls the pin on
the fire extinguisher. The next appropriate action in the use of the fire extinguisher is to

(1) Squeeze a handle on the extinguisher

(2) Sweep the fire from side to side with the extinguisher

(3) Aim at the base of the fire

(4) Sweep the fire from top to bottom with the extinguisher

The correct answer is Choice (3). Remember PASS. The nurse has already pulled the pin, so the next
action is to aim at the base of the fire. Then she’ll squeeze the handle of the extinguisher and extinguish
the fire by sweeping from side to side.
An adult has been treated for pulmonary tuberculosis (TB) and is being discharged home with his wife
and two children. His wife asks how TB is passed from one person to another. To help the wife prevent
anyone else from catching it, how should the nurse respond?

(1) “You should wear gloves when handling his linen and bedding because you can get tuberculosis by
touching the germs.”

(2) “You should keep the windows and doors closed so as not to spread the droplets.”

(3) “He must be careful to cough into a handkerchief that’s washed in hot water or discarded after use.”

(4) “Make sure to boil all milk before drinking or using it.”

The correct answer is Choice (3). TB is transmitted by airborne droplets, specifically the airborne droplet
nuclei, so the answer you’re looking for limits access to these droplets. Tuberculosis may remain in the
air for long periods of time. Thus, care should be taken when coughing or sneezing to limit the spread of
residue.

A nurse is caring for a patient with an internal radiation implant. Which of the following is an
inappropriate component for the nurse to include in the plan of care?

(1) Place the client in a semiprivate room at the end of the hall.

(2) Wear gloves when emptying the client’s bedpan.

(3) Keep all linens in the room until the implant is removed.

(4) Wear a lead apron when providing direct care to the patient.

The correct answer is Choice (1). Think about the appropriate actions to take for a patient with internal
radiation implants to help you find the inappropriate choice. You’d wear gloves when emptying the
client’s bedpan, keep linens in the room until the implant is removed, and wear a lead apron when
providing care (to protect yourself). That leaves Choice (1). A private room with a private bath is
essential because it prevents accidental exposure of other patients or healthcare workers to the
radiation.

A pregnant woman presents in the emergency room with complaints of severe headache for two days,
some episodes of double vision, and an observation that her rings have become tight lately. She has a
history of hypertension and is on a sodium-restricted diet. Which of the following assessments would
the nurse report to the physician immediately?

(1) BP 130/88

(2) Proteinuria 2+ on dipstick

(3) Fetal heart rate of 146 with good variability


(4) 2+ nonpitting edema of the feet

Choice (1) isn’t an abnormal finding based on standard normal blood pressure values in adults of 100/70
to 140/90. Choice (2) is abnormal because urine should never contain protein; it’s an indication that the
renal function is impaired. Choice (3) is normal; fetal heart rates range from 120 to 160 beats per
minute. And as for Choice (4), nonpitting edema of the feet in pregnancy is a common discomfort
related to increased pressure on the venous return by the growing uterus. With this information
gathered from thoroughly examining each answer option, you can easily decide which finding should be
reported to the physician. Choice (2) is correct because it indicates abnormal renal function needing
immediate evaluation and treatment. Isn’t this easy? Or at least easier than you thought it was going to
be? Regardless of the primary diagnosis of this patient, the abnormal finding impacts her well-being the
same way. Don’t let the fact that questions contain integrated content distract you from what you
already know!

A patient has just been brought to the emergency department by ambulance. The patient has stated
that he has been vomiting a large amount of blood for several days. Which of the following actions
would be the nurse’s first priority?

(1) Take vital signs.

(2) Give the patient a drink of water.

(3) Give an antiemetic.

(4) Schedule the patient for surgery immediately.

The correct answer is Choice (1). The keywords here are vomiting a large amount of blood, several days,
and nurse’s first priority. In this example, you must know about blood loss and then apply the
information to the question scenario. Knowing that blood loss can cause all types of problems, you must
first assess vital signs to determine patient stability and assess for symptoms of shock.

A 58-year-old male is admitted to the emergency room with a blood sugar level of 640 mg/ dL. The
nurse assesses the patient for the following: fruity breath, Kussmaul respirations, and lethargy. Family
arrives and tells the nurse that the patient was just recently diagnosed with type 2 diabetes mellitus and
hasn’t been following the recommended diet or medication regime. Which of the following
interventions would be the nurse’s next best steps in this situation? Select all that apply.

(1) Giving glucagon IV

(2) Starting an IV of D5W at 150 ml/hour

(3) Starting an IV infusion of regular insulin

(4) Starting an IV of normal saline at 150 ml/hour

(5) Drawing an ABG


The correct answers are Choices (3) and (4). Keywords are 58-year-old, blood sugar level of 640 mg/ dL,
fruity breath, Kussmaul respirations, lethargy, type 2 diabetes, hasn’t been following diet and
medication regime, and top two priorities. Choice (3) addresses the problem of bringing down high
blood glucose. Choice (4) deals with the fact the patient is dehydrated from all of the diuresis and
urinating. The patient has lost fluids and needs normal saline to replenish the fluid loss. Choice (1),
giving glucagon, makes the blood sugar rise, not come down. Choice (2) gives more glucose, and the
patient already has too much. Drawing an ABG, Choice (5), isn’t a priority in this situation.

When making rounds, a nurse finds the patient on the floor, where it appears he has fallen. What should
be the nurse’s initial response?

(1) Inspect the patient for injuries.

(2) Transfer the patient back to bed.

(3) Move the patient to a chair.

(4) Report the incident to a supervisor.

The correct answer is Choice (1). Keywords in this question are finds a patient on the floor and initial
response. The appears he has fallen isn’t important because that’s an assumption; the nurse doesn’t
know objectively how the patient got to the floor. You have to determine what the priority action for
finding a patient on the floor is. In an emergency situation, a nurse must first assess the condition of the
patient. This assessment is basic to any emergency response by a nurse. Moving a patient before doing a
complete assessment can make any injury worse. After you collect information through assessment, you
need to verify that it’s accurate. You can verify the information by collecting additional data, questioning
orders, obtaining judgments and/or conclusions from other team members when appropriate, and
collecting data yourself rather than relying on technology or other people. Verifying data ensures its
authenticity and accuracy.

A 22-year-old man is brought to the emergency department after sustaining a skull fracture in a motor
vehicle accident. Thirty minutes after his admission, the nursing assessment reveals a Glasgow Coma
Score decrease from 10 to 5, signs of nuchal rigidity, and changes in mental status. On the basis of the
nursing assessment, what is the priority nursing intervention?

(1) Notifying the physician of the client’s change in status

(2) Implementing measures to decrease intracranial pressure

(3) Minimizing environmental stimuli

(4) Protecting the client from injury if seizures occur

The keywords for this question are 22-year-old man, sustaining a skull fracture, Glasgow Coma Score
decrease from 10 to 5, nuchal rigidity, changes in mental status, and priority nursing intervention. The
correct answer is Choice (1). The client is exhibiting signs of an acute deterioration in neurologic status,
and the physician must be notified immediately. In this case, the nurse has already completed a focused
assessment and identified a potentially life-threatening emergency. The other options are interventions
and may be needed, but the priority is to contact the physician.

A patient is brought into an emergency department complaining of chest pressure. His blood pressure is
160/98 with a pulse of 94 and respirations of 22. A nurse administers nitroglycerin 0.4 mg sublingual as
ordered. Reevaluation in five minutes reveals that the patient’s blood pressure is 100/72 with a pulse of
94 and respirations of 20. What should the nurse do next?

(1) Note that the patient has become hypotensive and obtain an order to administer fluid.

(2) Place the patient in semi-Fowler’s position and administer oxygen at 4 L per minute.

(3) Administer a second dose of nitroglycerin.

(4) Document the results and continue to monitor the patient.

The correct answer is Choice (4). The keywords here are complaining of chest pressure, blood pressure
is 160/98, pulse of 94 and respirations of 22, nurse administers nitroglycerin, reevaluation in five
minutes, blood pressure is 100/72 with a pulse of 94 and respirations of 20, and nurse do next. If you
were to reword this question, it would say, “What do you do for a patient who has a normal response to
nitroglycerin?” The key to this question is knowing what the normal response to nitroglycerin is and
understanding how the medication works. Because the findings are normal, the other interventions
aren’t necessary.

The nurse is caring for an emergency room patient who was in a motor vehicle accident and is now in X-
ray. A phone call regarding the patient is transferred to the nurse. When the nurse answers, a woman
on the phone states she’s the patient’s wife and wants to know what happened and how her husband is
doing. Which of the following responses by the nurse would be most appropriate?

(1) “I know you’re upset, but I can’t give out information regarding any patient over the phone.”

(2) “I’ll transfer you to the doctor so that he can explain the extent of the injuries.”

(3) “The patient is currently in X-ray and is doing well.”

(4) “The patient will call you himself when he gets back from the X-ray.”

The correct answer is Choice (1). The keywords for this question are emergency room patient, motor
vehicle accident, in X-ray, woman on the phone states she’s the patient’s wife, and which of the
following responses by the nurse would be most appropriate? Although refusing to answer the
questions of a worried family member may seem heartless, sometimes the caller really isn’t a family
member; it may be an insurance agent, lawyer, or someone who may use the information to harm the
patient. Respecting a client’s confidentiality ensures that the patient receives safe care. In this case, the
nurse can tell the patient that his wife called, but telling the wife that the patient will call her when he
gets back from X-ray reveals too much information. Other healthcare providers are bound to the same
confidentiality clauses as nurses and aren’t permitted to share patient information either.
After a busy morning, the nurse is finally charting online in the hallway for her care of a client in room
204. The hallway is full of visitors for her new patient just a few steps away. An emergency light comes
on in another room. Which of the following actions should the nurse do next?

(1) Minimize the screen and go help the emergency client

(2) Turn the screen around toward the wall

(3) Leave the computer as it is and go

(4) Close all screens prior to answering the emergency call light

Keywords in this question are charting online in the hallway, hallway is full of visitors, emergency light
comes on, and which of the following actions should the nurse do next? The correct answer is Choice
(4). The nurse needs to close the computer screens and protect the information from any visitors
walking past. The other options leave visitors — or anyone — able to view the client’s information.

A victim of a major fire arrives at the emergency department. He has sustained a major burn injury.
Which of the following metabolic alterations is expected during the initial period post-burn?

(1) Hyponatremia and hypokalemia

(2) Hyponatremia and hyperkalemia

(3) Hypernatremia and hypokalemia

(4) Hypernatremia and hyperkalemia

Keywords in this question are a victim of a major fire, sustained a major burn injury, and which of the
following metabolic alterations is expected during the initial period post-burn. The correct answer is
Choice (2). When the cell membrane has burned, the contents of the cell spill out into the serum.
Potassium is the major electrolyte in a cell, so in this situation potassium floods the serum. With the loss
of fluid after a burn, sodium is lost.

A client comes into the emergency room with complaints of dizziness, nausea, and vomiting. The doctor
suspects Ménière’s disease. What is the nurse’s priority action?

(1) Give an antiemetic

(2) Initiate a sodium-restricted diet

(3) Give an antihistamine

(4) Initiate safety measures

The keywords in this question are emergency room; dizziness, nausea, and vomiting; Ménière’s disease;
and nurse’s priority action. The correct answer is Choice (4). A Ménière’s patient is at high risk for falls;
therefore, safety measures need to be put in place. The other answers are interventions for this
condition but aren’t a priority at this time.
A client is admitted to the emergency department with an active diagnosis of acute myocardial
infarction. The patient is started on tissue plasminogen activator (tPA) by infusion. Which of
the following parameters would a nurse determine requires the least frequent assessment to detect
complications of therapy with tPA?

(1) Oxygen saturation

(2) Neurological signs

(3) Blood pressure and pulse

(4) Complaints of abdominal and back pain

Keywords in this question are an active diagnosis of acute myocardial infarction, tissue plasminogen
activator, which of the following parameters requires the least frequent assessment, and detect
complications of therapy. The correct answer is Choice (1). An acute myocardial infarction is due to a
clot or plaque blockage, and the drug is given to break up clots. So, your priority is bleeding. The key
phrase here is least frequent assessment. Thrombolytic agents dissolve clots, and bleeding can occur
anywhere in the body. The nurse monitors for any signs of bleeding and also for occult (hidden) signs of
bleeding by taking hemoglobin and hematocrit values, blood pressure, and pulse; checking neurological
signs; assessing abdominal and back pain; and checking for the presence of blood in the stool and/or
urine. Bleeding is the primary complication of thrombolytic therapy.

1. A 75-year-old male has been admitted to the emergency department. Upon arrival, he is complaining
of dyspnea, peripheral edema, fatigue, and having only gone to the bathroom three times in the past 24
hours, producing a very small amount of urine each time. The history of the patient reveals that he has
type 1 diabetes and has a history of hypertension. He has just finished his antibiotic for sepsis yesterday.
Which of the following diagnostic tests would the nurse expect to see on the physician’s orders to
confirm the diagnosis of acute kidney injury? Select all that apply.

(1) Chest X-ray

(2) UA

(3) UA culture and sensitivity

(4) Ultrasound of the heart

(5) CT scan of the kidneys

2, 3, and 5.

The patient is an older adult at 75, which is a risk factor. He has a history of diabetes and
hypertension — both risk factors. He’s had an infection or sepsis and just finished the antibiotic for it,
which puts him at risk for acute kidney injury. He has the signs and symptoms of acute kidney injury
already: dyspnea, peripheral edema, fatigue, and decreased urine output over the past 24 hours. The
diagnostics test for acute kidney injury would be the UA, UA culture and sensitivity, and a CT scan of the
kidneys.
82 A patient has sustained a major electrical burn following an injury while working on a telephone pole.
What would be the nurse’s immediate concern on the patient’s arrival to the emergency department?

(1) The patient’s pain level

(2) The entry and exit wounds on the patient’s body

(3) Preventing the patient from moving his extremities

(4) Establishing the patient’s respiratory status

4.

Securing an airway is the first consideration following trauma. The other options are important but
secondary to breathing.

108 A 2-year-old child is in the emergency department for a crush injury of her right leg. Laboratory
studies indicate that her potassium level is 6.2 mEq/L. What clinical manifestation should the nurse be
especially alert to in this child?

(1) Laryngospasm

(2) Tetany

(3) Ventricular fibrillation

(4) Hyperthermia

3.

Crush injuries can result in serum potassium excess (hyperkalemia with K+ over 5.5 mEq/L), which can
result in ventricular fibrillation and cardiac arrest. Laryngospasm isn’t directly related to a crush injury of
the right leg, and tetany is a manifestation of hypocalcemia. Hyperthermia isn’t related to a crush injury
involving an extremity, but it may be related to an injury to the CNS.

124 A 2-year-old child has arrived in the emergency department after being removed from her burning
apartment. Her face is smoke-tinged, and her hair is singed. Which of these measures would be the
nurse’s first priority?

(1) Start an intravenous line and replace fluids.

(2) Establish and maintain an airway.

(3) Administer tetanus prophylaxis.

(4) Obtain baseline laboratory studies.

2.
Singed hair and a smoke-covered face or smoky breath indicates possible smoke inhalation with the
potential for upper airway edema. The first priority is to establish and maintain an airway. Establishing
an IV line and fluid replacement is the next priority. The need for administering tetanus prophylaxis
must be evaluated by the care provider, and obtaining baseline laboratory studies should be done
shortly after.

148 A nurse has called the doctor and stated the patient has an oncological emergency. Which of the
following are manifestations of an oncological emergency? Select all that apply.

(1) Calcium of 12.5

(2) Periorbital edema in the morning

(3) Spinal cord compression

(4) Diabetes insipidus

(5) Syndrome of inappropriate antidiuretic hormone (SIADH)

1, 2, 3, and 5.

Syndrome of inappropriate antidiuretic hormone, spinal cord compression, a calcium level of 12.5, and
periorbital edema in the morning are all manifestations of oncological emergencies. The only listed
choice that isn’t one is diabetes insipidus, Choice (4).

208 A woman is admitted to the emergency room with a pneumothorax. The nurse would expect the
patient to exhibit

(1) Breath sounds bilaterally

(2) Bradycardia

(3) Tracheal deviation

(4) Hypertension

3.

A pneumothorax patient could have a tracheal deviation to one side. She would have absent breath
sounds on the affected side and be hypotensive (not hypertensive) and tachycardic (not bradycardic).

223 A nurse in the emergency department is triaging a group of patients. Which patient should be seen
first?

(1) A 49-year-old male with a complex fracture of his right leg due to a motor vehicle accident who is
complaining of severe pain

(2) An 11-year-old male who is holding his thumb with a bloody towel and who reports that he cut it on
an old tin can
(3) A 78-year-old female who is complaining of difficulty speaking, left-sided weakness, and headache
for the past two hours

(4) A 14-year-old female who has epilepsy and who had a grand mal seizure an hour ago at the beach

3. This patient is demonstrating signs of an evolving cerebrovascular accident (CVA). Timing is important
in this case because she’s only had symptoms for two hours. This patient may benefit from the
administration of recombinant tissue plasminogen (tPA), which can lyse a thrombotic clot, thus
preventing cell death in brain tissue. However, for maximum effect, this medication must be given
within three hours of onset of symptoms. The other patients are in need of attention, especially the
patient with the complex fracture, but the patient with the potential evolving CVA takes priority.

242 A young woman is being assessed in the emergency department following her report of being raped
by a neighbor in her home. The patient states, “I shouldn’t have opened the door; my parents will kill
me.” The nurse’s best response is

(1) “You should never open the door when you’re home alone.”

(2) “Don’t worry, your parents will have to understand.”

(3) “This must be very difficult for you.”

(4) “You shouldn’t worry about this now.”

3. This response by the nurse demonstrates empathy and reflects the patient’s feelings; it opens
channels of communication. Admonishing the woman about opening the door just reinforces guilt and
negative feelings. Telling the woman not to worry offers reassurances that may not be real because the
nurse has no way of knowing the parents’ reactions. Telling the woman not to think about what
happened dismisses the woman’s anxiety and pain.

Reference: Rhoda Sommer - NCLEX-RN For Dummies, 2 nd edition (2020)

75. In triage of a disaster event, the civilian priority is care to the victims who:

1. Are most likely to survive.

2. Have children.

3. Have the least serious wounds and can be cared for quickly.

4. Have the most serious, life-threatening injuries.

Comprehension, assessment, physiological integrity, (a). 4. The civilian priority is for the care of patients
whose lives are threatened. 1. The military priority is for those who are most likely to survive. 2. Triage is
done on the basis of risk to the patient, not to survivors or family members. 3. This choice places people
already at the greatest risk in further danger of morbidity or mortality.
64 The nurse just received a call from the emergency department that a patient diagnosed with
acquired immunodeficiency syndrome (AIDS) is coming to the unit. Which action should the nurse take
first?

1. Prepare the patient’s room for isolation.

2. Face her own feelings about AIDS.

3. Orient the patient to his or her room.

4. Explain unit policies to the patient.

2. The nurse must be able to deal with his or her own feelings first. 1. Standard Precautions are all that
are required. 3. This is not a priority. 4. This is not the first thing to do.

65 A 16-year-old patient is admitted to the neurology floor after being involved in a motor vehicle
accident (MVA). His head hit the windshield, and he is being admitted for observation. During the
afternoon he begins to complain of a headache, has two episodes of vomiting, and is more difficult to
arouse. The initial nursing intervention is to:

1. Do nothing; he needs his rest.

2. Place him in a recumbent position, administer oxygen, and notify the physician immediately.

3. Prepare him for emergency surgery.

4. Assess his neurological status, elevate the head of the bed slightly, and notify the physician
immediately.

4. A change in neurological status may indicate increased intracranial pressure (IICP). A thorough
neurological assessment is indicated. Placing the head of bed in a slightly elevated position aids in
decreasing the pressure rise. The physician should be notified immediately in case emergency
interventions are required. 1. This is an inappropriate and negligent action. 2. In suspected IICP, the
head of the bed is elevated slightly to decrease intracranial pressure. 3. Preparing the patient for
emergency surgery may be indicated, but the nurse must begin with assessment of the patient’s status
and subsequent notification of the physician.

77 A 12-year-old patient is in the emergency department with a dislocated shoulder after a playground
accident. The physician orders midazolam (Versed), 0.5 mg IV, before performing a closed reduction of
the shoulder. Which nursing diagnosis is a priority for this patient?

1. Ineffective airway clearance

2. Pain related to trauma

3. Ineffective health maintenance


4. Risk for infection related to trauma

1. The effect of conscious sedation requires careful monitoring of the airway; therefore the risk for
ineffective airway clearance is appropriate. 2. Although a concern and an appropriate nursing diagnosis
for the patient, it is not a priority. 3. This nursing diagnosis is appropriate after the procedure when
discussing physical limitations with the patient. 4. This is inappropriate because the procedure being
done is noninvasive.

89 A nurse arrives at the scene of an accident and finds a man lying on the ground with his eyes closed.
The nurse’s first action should be to:

1. Notify emergency personnel.

2. Open the airway with the head tilt–chin lift maneuver.

3. Attempt to arouse the person.

4. Start cardiopulmonary resuscitation.

3. Assessment begins with first determining responsiveness. 1. Activation of the emergency medical
system occurs after responsiveness is determined. 2, 4. These actions are initiated only after
determining responsiveness.

94 A competitive racer had a near-drowning accident when the canoe she was paddling tipped over and
struck her on the head. Her lips and nails are cyanotic. Immediate emergency care for this patient’s
hypoxia includes:

1. Bag-valve-mask resuscitation.

2. Inserting chest tubes to drain the water.

3. Cricothyroid puncture to ensure a patent airway.

4. Placing her in Trendelenburg position to facilitate fluid drainage.

1. Bag-valve-mask or Ambu ventilation provides oxygen to correct the hypoxia. 2. Chest tubes only
drain the pleural space. 3. Cricothyroid puncture would be indicated if injury to the upper airway has
occurred. 4. Trendelenburg position may assist with fluid drainage but does not help to correct hypoxia;
it may also cause dyspnea.

108 A 42-year-old man has ingested 18 diazepam (Valium) tablets and 18 unidentified capsules. He is in
the emergency department and is now alert after gastric lavage. He is calling his son to come and take
him home. He is exhibiting which defense mechanisms?

1. Denial

2. Projection

3. Regression
4. Sublimation

1. Denial is indicated by the patient’s inability to recognize the seriousness of his act and his belief
that he can go home. 2. Projection is placing unacceptable impulses onto another person. 3. Regression
is moving back to an earlier time or developmental level during periods of stress. 4. Sublimation is the
process by which negative impulses are channeled into more acceptable outlets.

112 A 21-year-old unconscious man is brought to the emergency department by a friend. His breath
smells of alcohol, and the friend reports that they were at a fraternity party where everyone was
drinking quite heavily. The nurse should:

1. Treat the patient for shock.

2. Ensure adequate airway and ventilation.

3. Recommend rest in a quiet environment.

4. Administer disulfiram (Antabuse) immediately.

2. Vomiting from acute alcoholism can cause aspiration, and the depressed central nervous system
can lead to respiratory arrest. 1. Shock is associated with delirium tremens, not acute alcoholism. 3.
This is appropriate for mild alcohol withdrawal; acute alcoholism requires observation. 4. Disulfiram may
be recommended as part of an aftercare program for the person with alcoholism who has already
undergone withdrawal.

125 While the nurse’s 14-year-old neighbor boy is mowing her lawn, he is stung by a bee. He starts
wheezing and complaining of difficulty breathing. The nurse takes him to the emergency department,
where he is diagnosed as having:

1. Asthma.

2. Hay fever.

3. Pneumonia.

4. Anaphylaxis.

4. Wheezing and respiratory distress are symptoms of an anaphylactic reaction to the bee sting. 1, 2,
3. Although wheezing and respiratory distress are common with these disorders, the precipitating event
was the bee sting.

48 A conscious person with diabetes arrives at the emergency department with a blood glucose level of
378 mg/dL. The nurse should expect the treatment of choice would be to:

1. Administer regular insulin.

2. Administer 50% glucose intravenously.


3. Initiate an intravenous (IV) dextrose infusion.

4. Monitor the potassium level.

1. Regular insulin is administered either intravenously or subcutaneously to treat the elevated blood
glucose. 2. This is the recommended treatment for hypoglycemia. 3. An IV infusion should be initiated
with a normal saline solution, not a dextrose solution. 4. Blood potassium levels should be monitored,
but this is not a treatment.

49 When a patient in diabetic ketoacidosis comes to the emergency department, blood electrolyte levels
should be monitored. The nurse is aware that the most important electrolyte to monitor in this patient
is:

1. Calcium.

2. Chloride.

3. Potassium.

4. Sodium.

3. Potassium is needed to allow the body to use insulin to metabolize glucose. 1, 2, 4. These
electrolytes do not work with metabolism of glucose and insulin.

50 What would be a priority emergency nursing concern for a victim with severe burns?

1. Relieving pain

2. Tetanus prophylaxis

3. Hemodynamic stability

4. Administering cardiopulmonary resuscitation

3. The hemodynamic instability that follows burn injury must be stopped to proceed with airway,
breathing, and circulation and prevent further trauma to the victim. 1, 2. These are appropriate but
not priorities. 4. The situation does not indicate that CPR is necessary.

91 Immediate physical observations of a burn patient by emergency personnel would include which of
the following as the most important?

1. Quality of respirations

2. Physical development

3. Height and weight

4. Head circumference
1. An immediate physical observation of any burn patient, especially a person burned in the area of
the face, is assessment of respiration (difficulty, rate, sound). 2, 3, 4. These are not immediate physical
observations during an emergency.

118 A patient is in the emergency department after an automobile accident. Which problem would the
nurse consider the most emergent?

1. The blood glucose level is high at 396 mg/dL.

2. The patient has a 3-cm laceration of his left eye.

3. The patient is complaining of shortness of breath.

4. A distraught family member is crying in the hallway.

3. This complaint can be life-threatening and is a priority. 1. This blood glucose level does need to be
corrected; however, respiratory status takes priority. 2. This small laceration needs to be attended to at
some time; however, the shortness of breath is of primary importance. 4. Although it is important to
keep the family calm and informed, the patient’s complaints of “shortness of breath” are priority.

4 A patient is brought to the emergency department after falling from a ladder and sustaining a fracture
of the left leg. The nurse observes that the skin is broken and part of the bone fragment is visible. The
appropriate intervention is to:

1. Immobilize the leg and cover the open wound.

2. Apply traction to the leg to attempt to reduce the fracture.

3. Elevate the leg as high as possible with pillows.

4. Apply an abduction pillow between the patient’s knees.

1. The bone ends may cause further injury to the surrounding tissue, and this patient has an increased
risk of infection. 2. Reduction of the fracture will be undertaken later, possibly in surgery. 3. Moving the
leg to elevate it can cause further injury. 4. Proper immobilization would require the use of splints above
and below the fracture site.

5 The nurse receives a telephone call from a neighbor whose husband collapsed while mowing the lawn.
He is described as being dizzy, confused, clammy, and nauseated. The nurse should instruct the wife to:

1. Encourage the patient to ambulate vigorously to stimulate circulation.

2. Have the patient drink saltwater to replace fluids lost through vomiting and diaphoresis.

3. Leave the patient in the position in which he was found and apply blankets.

4. Move the patient to a cool, quiet area, and loosen his clothing.
4. The patient should be removed from the heat and made as comfortable as possible to reduce
further illness caused by heat and exertion. 1, 3. The patient should not exert himself or be subjected
to any more heat. 2. If the patient is vomiting, nothing should be given by mouth, to prevent aspiration
or gastric distention.

6 A man who was rescued from drowning was brought to the emergency department for follow-up. The
nurse should assess and observe for:

1. Pulmonary edema.

2. Ketoacidosis.

3. Fluid and electrolyte imbalance.

4. Left shift.

1. The patient is at risk of developing pulmonary edema as a result of the trauma of near drowning. 2.
Alteration in glucose metabolism is less likely to occur compared with the risk of respiratory distress and
pulmonary edema. 3. Alteration in fluid imbalance is less likely to occur compared with the risk of
respiratory distress and pulmonary edema. 4. Alterations in blood counts are less likely to occur
compared with the risk of respiratory distress and pulmonary edema.

8 A firefighter who was brought to the emergency department for evaluation after an accident in a
factory insists that he is not injured and wants to wait until other victims are treated before being seen
by the physician. The nurse observes that he has singed nasal hair and a brassy cough and recognizes
that he is at risk now for:

1. Pulmonary edema as a result of smoke inhalation.

2. Respiratory infection caused by exposure to toxic chemicals.

3. Esophageal reflux as a result of irritation of the oropharynx.

4. Increased sputum production related to oxygen deprivation.

1. Smoke inhalation, as evidenced by brassy cough, singed nasal hairs, and respiratory distress, is a
serious complication that can lead to death if left untreated. 2. Infection is not an immediate risk. 3.
Gastric distress is not an immediate risk. 4. Sputum production is not related to oxygen deprivation.

9 A marathoner comes to the emergency department reporting the sudden onset of sharp chest pain
and difficulty breathing during his usual daily workout. Chest excursion on the affected side is reduced.
The nurse assesses him for:

1. Muscle strain.

2. Respiratory infection.

3. Spontaneous pneumothorax.
4. Myocardial infarction.

3. Spontaneous pneumothorax can occur during periods of strenuous activity. 1. Muscle strain does
not commonly cause sudden pain in the chest. 2. Infection does not commonly cause sudden pain in the
chest. 4. Myocardial infarction does not typically produce alterations in chest excursion.

11 An individual arrives in the emergency room after having been found in hypovolemic shock.
Emergency medical technicians (EMTs) arrive at the scene and intubate the patient, start an intravenous
infusion, and obtain an electrocardiogram (ECG). The receiving nurse in the emergency department
knows that one of the first tests to be performed is/are which of the following?

1. Complete blood count (CBC) to rule out leukopenia.

2. Liver function tests (LFTs) to rule out cirrhosis.

3. Hemoglobin and hematocrit (H&H) and electrolytes to rule out anemia and dehydration.

4. Glucose to rule out hypoglycemia.

3. Hemoglobin and hematocrit and electrolytes can identify severe anemia and hypernatremia, which
can cause hypovolemic shock. 1, 2, 4. These are not causes of hypovolemic shock.

12 A patient who has been in a motorcycle accident states that he is experiencing pain in his neck,
numbness and tingling in his legs, and weakness on his left side. The nurse should:

1. Remove his helmet as quickly as possible.

2. Ask him to remove his helmet.

3. Leave his helmet in place until assistance is available to remove it.

4. Secure the helmet in place with adhesive tape.

3. To reduce risk of further injury to the patient’s cervical spine, the helmet should be removed only
when assistance is available to stabilize his neck. 1, 2, 4. These actions would increase the risk of
further injury to the patient.

14 A nurse sees a motorcycle rider struck by a car and stops at the scene. The patient is alert, complains
of inability to move his legs, and has his helmet still in place. The nurse’s next action should be to:

1. Assess the patient and call 911.

2. Elevate his legs with his helmet after carefully removing it from his head.

3. Refrain from helping because of legal concerns.

4. Begin CPR.
1. Rapid entry into the emergency response system is indicated. 2. The helmet should not be removed,
and the legs should not be moved until qualified medical assistance with proper equipment arrives on
the scene. 3. Rapid entry into the emergency response system is delayed. 4. If the patient is awake and
talking, resuscitation is not a requirement.

16 A motor vehicle accident (MVA) victim has a hoarse voice, subcutaneous emphysema, cough with
hemoptysis, and respiratory distress. The nurse should prepare him for an immediate:

1. Blood transfusion.

2. Computed tomography or magnetic resonance imaging scan.

3. Cricothyrotomy or tracheostomy.

4. Intubation.

3. The patient with a fractured larynx must be prepared for immediate surgical intervention to
establish an airway. 1. Blood loss is a secondary risk in comparison with the compromised airway. 2.
Without a patent airway the patient would likely have a respiratory arrest during a radiological
assessment examination. 4. Because of the injuries to the neck, establishing an airway through
traditional vertical access most likely is not possible.

18 While shopping for groceries, the nurse witnesses an adult shopper fall to the ground. The patient is
assessed and found to be without pulse or respirations. The nurse begins CPR by placing his hands on
the sternum to begin compressions at a depth of:

1. Approximately 1 inch.

2. Approximately inches.

3. Approximately 2 inches.

4. Minimum of 2 inches.

4. This is the correct depth of compressions for an adult victim according to the American Heart
Association guidelines. 1. This is an inappropriate depth for any compressions in any age group. 2. This
is the correct depth for compressions in an infant. 3. This is the correct depth for compressions in a
child.

19 A 36-year-old man is brought to the emergency department with a diagnosis of morphine overdose.
The first medication the nurse would expect to administer in his treatment should be:

1. Disulfiram (Antabuse).

2. Epinephrine (Adrenalin).

3. Methylprednisolone (Solu-Medrol).

4. Naloxone (Narcan).
4. Naloxone is a narcotic antagonist and reverses the adverse effects of an overdose. 1. Disulfiram is
prescribed for alcohol dependency. 2. Epinephrine is not indicated for initial treatment of narcotic
overdose. 3. Methylprednisolone is not indicated for initial treatment of narcotic overdose.

22 A man is brought to the hospital after having been in a house fire. He is burned over all of his chest
and his right leg. Using the rule of nines, the nurse estimates that the burned percentage of his body is:

1. 25% to 30%.

2. 1% to 5%.

3. 0% to 15%.

4. 75% to 80%.

1. The chest is estimated at 18% of the body, and the leg is estimated at 9%; therefore, the combined
area would be approximately 27%. 2. This percentage is equal to less than the area of one arm. 3. This
percentage is approximately equal to the area of one arm. 4. This is a much larger area than is indicated
on the patient’s chart.

23 A patient arrives in the emergency department with a fractured left wrist. The complaint that is the
most likely to indicate serious injury and must be addressed first is:

1. Pain in the affected wrist.

2. Swelling in the fingers.

3. Bruising in the hand and arm.

4. Numbness in the fingers.

4. Numbness in the fingers can signal nerve damage. 1. Pain is an expected symptom in a fracture. 2.
Swelling is an expected symptom in a fracture. 3. Bruising is a common symptom in a fracture.

24 A firefighter is brought to the hospital by ambulance from the scene of a factory explosion where he
was burned and also sustained a severe laceration to his lower right leg. His heart rate is 112 beats/min,
respirations are 78 and shallow, and blood pressure is 72/40 mm Hg. The nurse recognizes that the
patient is in:

1. Hypovolemic shock.

2. Neurogenic shock.

3. Septic shock.

4. Cardiogenic shock.
1. Hypovolemic shock is primarily a fluid problem caused by loss of blood or fluid volume. 2.
Neurogenic shock results from widespread dilation of blood vessels caused by an imbalance in
autonomic stimulation of the smooth muscles in vessel walls. It is usually associated with spinal cord
injury. 3. Septic shock results from complications of massive bacterial infection in which toxins released
into the blood cause vasodilation. 4. Cardiogenic shock is the result of faulty pumping action, an unlikely
condition in a firefighter in good physical condition.

29 A hiker is brought to the emergency department after having been bitten on his lower leg by a snake.
The nurse immobilizes the affected limb and:

1. Places it in a dependent position.

2. Elevates it on two pillows.

3. Places warm moist packs on it.

4. Cleans it vigorously with antimicrobial soap.

1. The affected limb is placed in a dependent position to minimize circulation of toxins. 2. Elevation
would facilitate circulation of the toxin. 3. Heat and increased circulation would facilitate circulation of
the toxin. 4. Cleaning and increased circulation would facilitate circulation of the toxin.

30 A spill of powdered chemicals occurs at a factory, and one of the employees is contaminated with the
substance. Before he is transported to the emergency department, the contaminant should be:

1. Flushed from his skin with running water for at least 10 minutes.

2. Contained by applying dry dressings to the affected skin.

3. Swept from the skin with a dry dressing.

4. Neutralized with petroleum jelly.

3. The powder should be carefully swept from the skin, making sure not to contaminate anyone else
in the process. 1. Diluting with water increases the volume of the contaminant and increases the risk of
tissue injury. 2. The contaminant should be removed from the skin immediately. 4. Petroleum jelly is
ineffective in neutralizing powdered chemicals.

31 A man who was involved in an MVA insists that he is “only bruised” and does not want to be
admitted to the hospital for observation. The nurse observes that he has decreased chest wall stability
and suspects several fractured ribs. She tells him:

1. “You can go home if you promise to come back if you start having trouble breathing.”

2. “You may have injuries to your lungs or heart as a result of rib fractures. Staying here would provide
you with critical immediate treatment should problems with pain or breathing occur.”

3. “You probably don’t have anything seriously wrong, but it would be a good idea to stay anyway.”
4. “You will have to sign this form stating that you are leaving against medical advice.”

2. The patient has a flail chest and may have cardiorespiratory problems as a result. Explaining the
risks to him increases his understanding of the situation and makes his compliance with admission
more likely. 1. If the patient develops respiratory difficulty, he may not be able to return to the hospital
in time to receive assistance. 3. The nurse should not dismiss the risks of complications and must inform
the patient of the situation and its possible outcomes. 4. The nurse must inform the patient of the risks
involved and not encourage him to leave against medical advice.

32 A patient who was severely injured in a fall from a scaffold is concerned about how much of his
independent function he will be able to regain. The nurse knows that his injury is at the T7 level. She
would expect that the patient may:

1. Be ventilator dependent for life.

2. Have paraplegia and partial loss of independence.

3. Not have any problems because the injury is so low in the spinal cord.

4. Have flaccid paralysis in all extremities.

2. This injury is often characterized by paraplegia, but the individual may be able to rehabilitate to
walk with support. 1. This outcome is characteristic of an injury at the cervical spine level. 3. Advising
the patient that he will have no problems in the case of any spinal injury is unrealistic. 4. This outcome is
characteristic of injury at higher levels (cervical and upper thoracic spine).

34 A 10-year-old boy is stabbed on the school playground. By the time the school nurse reaches him, he
is pale, cyanotic, and diaphoretic and has a pulse of 120 beats/min. The nurse’s highest priority action is
to:

1. Administer fluid boluses to maintain blood pressure.

2. Cover him with a blanket or jacket to maintain his body temperature.

3. Establish and maintain a patent airway.

4. Apply pressure to the bleeding wound.

3. Establishing and maintaining an airway are the highest priorities. 1. Fluid balance is a lesser priority
compared with maintaining the airway. 2. Body temperature is a lesser priority compared with
maintaining the airway. 4. This intervention, although appropriate for treatment of a penetrating
wound, is a lesser priority compared with maintaining the airway.

35 A 3-year-old boy was bitten by another child at his day care center. In giving discharge instructions,
the nurse includes information about:

1. Contacting an attorney.
2. Administering the full dose of the prescribed antibiotic.

3. Waking the child every hour through the night to make sure he can be roused.

4. Making sure that the child has a fluid intake of at least 700 mL over the next 24 hours.

2. To treat or prevent infection in the wound, all of the antibiotic must be administered as ordered. 1.
Discussing legal matters with the parents regarding the event is inappropriate for the nurse. 3. Checking
levels of consciousness is indicated for head injuries. 4. Forcing fluids is not indicated for treatment of
human bites.

37 A 36-year-old sexual assault survivor is brought to the emergency department for examination and
treatment. For the prevention of pregnancy, the nurse prepares to administer:

1. Cefixime (Suprax).

2. Norethindrone and mestranol (Ortho-Novum).

3. Estrogen and progesterone (Ovral).

4. Doxycycline (Vibramycin).

3. This medication is used after sexual contact to prevent pregnancy. 2. This contraceptive is prescribed
for administration on an ongoing basis before sexual contact. 1, 4. These are antibiotics that have no
contraceptive activity.

38 The police brought a 43-year-old man who is having tremors, intestinal cramps, chills, and sweating
to the emergency department. His eyes are watery, and his nose is runny. The nurse suspects:

1. Overdose of narcotics.

2. Withdrawal from narcotics.

3. Overdose of stimulants.

4. Withdrawal from stimulants.

2. Watery eyes and a runny nose are characteristic of narcotic withdrawal. 1. Overdose of narcotics
does not produce chills, sweating, or intestinal cramps. In addition, the patient would most likely be
comatose. 3, 4. Stimulants do not produce these symptoms, either in overdose or withdrawal.

39 A 2-year-old boy is brought to the emergency department after a bee sting. He is wheezing and has
urticaria and diffuse erythema. The nurse, recognizing that an allergic reaction can progress rapidly,
should:

1. Ensure adequate airway and ventilation.

2. Apply cold packs to the affected area for comfort.


3. Administer antibiotics to prevent infection.

4. Administer steroids and antihistamines to counteract swelling.

1. Anaphylactic shock can easily progress to respiratory arrest or compromised airway. The priority
actions are to establish and maintain the airway. 2. Although the application of cold may provide
comfort and decrease swelling, it is not a priority action. 3. Administration of antibiotics is a lesser
priority compared with establishing and maintaining an airway. 4. Administration of medications is a
lesser priority compared with establishing and maintaining an airway.

41 A 36-year-old woman receiving antibiotic therapy for a gram-negative infection comes to the
emergency department complaining of weakness, restlessness, and thirst. The vital sign assessment
reveals a temperature of 103.8 ° F (39.9 ° C). The nurse recognizes that this patient is likely experiencing:

1. Anaphylactic shock

2. Neurogenic shock.

3. Septic shock.

4. Anxiety reaction.

3. Septic shock results from the release of endotoxin that causes vasodilation. 1. Although the patient
might be allergic to antibiotics, a reaction would not be characterized by these symptoms. 2. Neurogenic
shock is the result of a decrease in circulating blood volume caused by an injury such as spinal cord
trauma. 4. The patient’s fever indicates that her experience is not psychogenic.

42 Assessment of a trauma patient reveals that his chest moves inward with inhalation and outward
with exhalation. The nurse recognizes this as a sign of:

1. Flail chest. 2. Pulmonary embolus. 3. Simple rib fracture. 4. Tension pneumothorax.

1. Paradoxical breathing results from instability of the chest wall in flail chest. 2. Pulmonary embolus is
characterized by sudden chest pain, dyspnea, and cyanosis. 3. Symptoms of a simple rib fracture include
pain on inspiration and rapid, shallow breathing. 4. Tension pneumothorax is characterized by neck vein
distention, tracheal deviation, and distant breath sounds on the affected side, in addition to paradoxical
breathing.

44 A trauma patient with an abdominal stab wound is brought by ambulance to the emergency
department. Inspection reveals that a loop of bowel is protruding from the wound. The nurse’s priority
action is to:

1. Administer oxygen by nasal mask. 2. Administer tetanus toxoid injection. 3. Apply wet dressings to the
wound site. 4. Apply a pressure dressing to the wound after replacing the bowel.
3. Tissue death takes place if the exposed bowel dries. 1. This is not a priority action for exposed bowel
tissue. 2. After ascertaining that the patient has had a primary tetanus immunization, the nurse should
prepare to administer a booster. However, this is not a priority action for exposed bowel tissue. 4. The
bowel should be replaced by a physician, possibly in a surgical procedure; this is not within the scope of
the nurse’s practice.

47 After an earthquake the victim who would be triaged with the highest priority is a(n):

1. Crying 4-year-old boy with a bleeding head laceration. 2. Dazed 26-year-old woman with a broken
right arm. 3. Unresponsive 48-year-old man with contusions of the head. 4. Alert 70-year-old woman
with diabetes who has burns of both arms.

3. The unconscious victim must be monitored for airway management. 1. The crying child appears to
have superficial injuries and can be treated later. 2. The fractured arm is not as much of a treatment
priority as is the head injury with loss of consciousness. 4. Although the age and underlying disease state
of this patient make her a high risk for complications, this is not as high a priority as is the head injury
with loss of consciousness.

48 A 52-year-old dockworker comes to the emergency department complaining of severe crushing chest
pain radiating to his left arm. He is cool, pale, and diaphoretic. The nurse recognizes that these might be
signs of a myocardial infarction. The first intervention would be to:

1. Apply electrodes for the cardiac monitor. 2. Administer nitroglycerin sublingually. 3. Administer
oxygen at 5 L/min by mask. 4. Insert a large-bore IV line and begin a normal saline infusion.

3. Immediate administration of oxygen may prevent more severe pain and tissue damage. 1. Comfort
and preservation of tissue integrity are higher priorities. 2. Further assessment must be done before
administering medication. 4. Comfort and preservation of tissue integrity are higher priorities.

49 The victim of a motor vehicle accident is brought to the emergency department and diagnosed with
an unstable fracture of the pelvis. The nurse should report to the physician the presence of:

1. Nausea and vomiting. 2. External rotation of the lower extremities. 3. Pain in the lower extremities. 4.
Increasing tenderness over the symphysis pubis.

4. Hemorrhage related to internal injuries can produce life-threatening complications in the patient
with a pelvic fracture. 1. Nausea and vomiting are common in patients who have pain and injury. 2.
External rotation is characteristic of pelvic fracture. 3. Pain in the back and lower extremities is common
in patients with pelvic fracture.

50 A construction worker without a hardhat on was injured when he was hit on the head by falling
lumber. He is bleeding profusely from the laceration on his scalp. The nurse’s priority is to:
1. Flush the wound with normal saline until it appears to be free of debris. 2. Scrub the wound with 4 × 4
gauze dressings and antiseptic skin cleanser. 3. Control bleeding and cover the open wound with a clean,
nonadhesive dressing until it is examined by a physician. 4. Apply antibiotic ointment to the wound and
apply a pressure dressing.

3. The wound should not be cleaned until after it has been examined by a physician who then treats it
accordingly. 1. Flushing the wound can cause alterations in its appearance or damage tissue by
dislodging clots that have formed. 2. Scrubbing would change the appearance of the wound and might
cause tissue damage and further bleeding. 4. Applying ointment would change the appearance of the
wound.

54 On the way to work a nurse encounters a motor vehicle accident with multiple trauma victims. The
nurse would know to provide immediate care to the individual experiencing:

1. Hemorrhage. 2. Altered level of consciousness. 3. Cyanosis of nail beds. 4. Obvious fracture of the
femur.

3. Cyanosis is a distinct sign of compromised oxygenation and is the highest priority of the situations
listed. 1. Hemorrhage may produce compromised oxygenation but is a lesser priority than cyanosis. 2.
Altered level of consciousness is less lifethreatening than cyanosis or hemorrhage. 4. This fracture is not
a life-threatening emergency.

55 A patient called 911 after falling down with a headache, dizziness, and nausea. The ambulance
personnel found him unconscious but breathing. They suspect that the victim was overcome by carbon
monoxide from a faulty furnace. Initial treatment for this patient may include oxygen administration at:

1. 2 L/min via nasal cannula. 2. 5 L/min via nasal cannula. 3. 40% by face mask. 4. 100% by face mask.

4. Saturating the blood with oxygen may allow the tissues in the body to remain oxygenated. 1. This
rate of flow does not provide adequate oxygenation. 2. This rate of flow allows carboxyhemoglobin to
remain in place. 3. This rate of flow does not break the affinity of hemoglobin for carbon monoxide.

57 To establish an airway in a patient with possible neck or spine injuries, the nurse uses the:

1. Head tilt–chin lift. 2. Head tilt–jaw thrust. 3. Chin lift only. 4. Jaw thrust only.

4. Tilting the head may cause further injury if the patient has trauma to the neck or spine. 1. This is the
first method to be used for a patient who does not have suspected neck or spine injury. 2. This is the
method to use in the case of a patient who does not have suspected neck or spine injury if additional
jaw displacement is required. 3. Lifting only the chin does not open the airway; it is likely to further
occlude it.
58 A patient who hit her head on the windshield in a motor vehicle accident has fixed, dilated pupils and
a positive Babinski sign. The nurse recognizes that these signs indicate:

1. Intracranial bleeding. 2. Internal bleeding. 3. Spinal cord injury. 4. Meningitis.

1. These are signs of a subdural hematoma. 2. Internal bleeding refers to the abdominal organs and
does not produce these symptoms. 3. Spinal cord injuries do not produce these symptoms. 4.
Meningitis, which would take several days to develop, does not produce these symptoms.

59 A lineman sustained electrical burns on both hands when he fell from a utility pole. His wife says that
his burns do not appear to be too serious. The nurse should explain to her why these burns are
considered to be:

1. Superficial. 2. Minor. 3. Moderate. 4. Major.

4. All electrical burns are considered to be major because of the possibility of internal damage to
tissue. 1. Superficial burns involve only the epidermal layer of tissue. 2. Minor burns involve 15% or less
of body surface area (BSA). 3. Moderate burns involve 15% to 25% of BSA.

60 In addition to inspecting the site of a penetrating wound, the nurse should also assess for a(n):

1. Foreign body such as a bullet. 2. Exit wound. 3. Medical information tag. 4. Concealed weapon.

2. Penetrating wounds often have an entrance and an exit and must be inspected thoroughly to assess
the extent of the trauma. 1. Internal inspection of a wound should be conducted by the physician. 3.
Information about conditions is a lesser priority compared with determining the extent of the injury. 4.
Although the patient may be armed, this information is a lesser priority compared with determining the
extent of the injury.

61 Which are considered to be major or critical burns? Select all that apply.

______1. Burns caused by electrical trauma ______2. Full-thickness burns that are less than 10% of
body surface area (BSA) ______3. Burns in patients with preexisting chronic conditions ______4. Partial-
thickness burns that are greater than 20% of BSA in children younger than 10 years ______5. Burns on
face, hands, or perineum

__X__ 1. Electrical burns can produce tissue damage based on conductivity through the body. _____ 2.
Full-thickness burns are considered major when they involve more than 10% of BSA. __X__ 3. Chronic
conditions such as diabetes or renal failure can cause lifethreatening complications for burn patients.
__X__ 4. Because of the ratio of body mass to BSA, the amount of involvement for partial-thickness
burns in children (20%) is less than in adults (25%). __X__ 5. Burns of the face, hands, or perineum are
more likely to produce complications compared with other areas of the body and are always considered
to be major injuries.
64 A patient caught the sleeve of her robe on fire while cooking breakfast. She is an 85-year-old woman
with insulin-dependent diabetes. She has superficial burns over 5% of her arm. This injury is classified as:

1. Minor. 2. Moderate. 3. Major. 4. Superficial.

3. Because of the age of the patient and her underlying chronic condition, this burn is considered a
major injury. 1, 2, 4. Although the burn is superficial, her age and underlying condition escalate the
classification of the burn.

65 A young man has been diagnosed with a strain to his left ankle sustained when he tripped on a rock
while he was jogging. The nurse should instruct him to do which of the following? Select all that apply.

______1. Place his foot on the floor to stabilize the ankle. ______2. Soak his ankle in warm water with
Epsom salts. ______3. Use crutches to avoid weight bearing. ______4. Exercise the foot to reduce the
chance for stiffening of the joints. ______5. Ice the affected area, and elevate.

_____ 1. The extremity should be elevated to reduce swelling. _____ 2. Ice is recommended to reduce
pain and swelling. Warm packs or soaks will increase swelling. __X__ 3. Treatment for sprains includes
immobilization and limited weight bearing. _____ 4. The foot should be immobilized until it has
recovered from the damage resulting from excessive physical use. __X__ 5. Ice is recommended to
reduce pain and swelling. Elevating the affected part will also aid in controlling edema.

66 What does the nurse focus on when assessing a recent fracture? Select all that apply.

______1. Pain ______2. Purulence ______3. Pallor ______4. Pulses ______5. Paresthesia

__X__ 1. The periosteum is richly supplied with nerve endings, producing significant pain in a fractured
bone. _____ 2. Purulence is a sign of infection, which would not be evident in the early stages of
recovery from a fracture. __X__ 3. Compromised circulation may be evidenced by pallor resulting from
impaired blood flow to the injured tissue. __X__ 4. Vascular compromise may result from compression
by bone fragments, swelling, or other tissue injury. Assessing for pulses on an ongoing basis is necessary
to detect changes as early as possible. __X__ 5. Paresthesia may result from compromise to the
neurological function of the injured tissue.

67 A patient who has been hemorrhaging from a severe injury is becoming restless, anxious, and
clammy. The nurse recognizes that this indicates possible progression to:

1. Shock. 2. Sepsis. 3. Sedation. 4. Syndactyly.

1. Restlessness, anxiety, and clammy skin are signs of shock in the bleeding patient. 2. Although sepsis
may produce shock in a bleeding patient, these are not symptoms of infection. 3. Sedation is produced
by the administration of medications. 4. Syndactyly is a congenital webbing of fingers or toes.
68 A 14-year-old football player arrives at the doctor’s office with his mother. The mother states she was
contacted by the coach because the teenager was tackled during practice and hit his head. In addition to
a neurological assessment, the nurse would expect the physician to:

1. Instruct the mother to wake her son every 2 hours to prevent concussion. 2. Administer ibuprofen for
pain control. 3. Instruct the patient and mother to avoid strenuous activity for 2 weeks, including
reading, playing video games, and watching television. 4. Administer Decadron to decrease risk of
intracranial swelling.

3. A concussion is a brain injury requiring rest and decreased activity to heal, including decreasing
activities that require concentration. 1. Waking an individual routinely will not change progression of an
injury. 2. Ibuprofen can promote bleeding, thus increasing risk of intracranial bleed. 4. Decadron would
not be administered without confirmation of intracranial swelling and increased intracranial pressure
(IICP).

69 A 34-year-old man was working in his garage when he accidentally impaled his hand on a
screwdriver. He was brought to the emergency room with a towel wrapped around his hand, and the
towel is now soaked in blood. The nurse should treat the wound by:

1. Removing the screwdriver immediately to prevent further tissue damage. 2. Removing the
screwdriver after flushing the area with normal saline for 5 minutes. 3. Leaving the screwdriver in place
to control bleeding. 4. Leaving the screwdriver in place as long as the patient is not in pain.

3. An impaled object should be left in place to control possible bleeding and to avoid further injury to
the tissue. 1. Removing the screwdriver can cause further trauma to the hand and severe bleeding. 2.
Cleaning the surrounding area does not diminish the possible damage that removing the screwdriver
might cause. 4. The patient is likely to experience pain either way, but removing the screwdriver can
cause further complications.

70 A 76-year-old man is admitted to the hospital with a cerebrovascular accident. The patient’s wife is
concerned because his daily aspirin regimen has not been included in his medication orders. As you
review the patient’s testing, the nurse should expect to find:

1. The patient had an ischemic stroke. 2. The patient had a hemorrhagic stroke. 3. The patient developed
a paralytic ileus. 4. The patient developed diplopia.

2. Aspirin is an antiplatelet that will increase the effects of a hemorrhagic stroke. 1. Antiplatelets may
be prescribed for an ischemic stroke to reduce the chance of recurrence. 3, 4. These diagnoses would
not influence the decision to discontinue aspirin therapy.

71 A patient with a tension pneumothorax complains of increased pain and dyspnea; his heart rate has
increased to 100 beats/min. The nurse recognizes that this may be the result of:

1. Coarctation of the aorta. 2. Cardiac tamponade. 3. Vasovagal response. 4. Mediastinal shift.


4. Mediastinal shift is a life-threatening condition that occurs when pressure within the intrapleural
space increases and the heart and mediastinal structures are pushed to the contralateral side. 1.
Coarctation of the aorta is a congenital condition characterized by stenosis of the vessel. 2. Cardiac
tamponade results from the accumulation of fluid in the pericardium. 3. When the vagus nerve is
activated, heart rate and blood pressure decrease.

72 An elderly woman who was brought to the emergency department from her nursing home
experienced sudden, sharp chest pain with shortness of breath and pallor. She is very anxious; her heart
rate is 112 beats/min, and her respiratory rate is 28 breaths/min. The nurse should explain to her that,
to confirm the physician’s diagnosis of possible pulmonary embolism, she will have a:

1. Chest x-ray examination. 2. Bronchoscopy. 3. Ventilation-perfusion scan. 4. Lung biopsy.

3. Nuclear scintigraphic / scanning of the lung is the diagnostic modality for detecting pulmonary
thromboembolism. 1. A chest x-ray examination would not necessarily demonstrate the pathology of a
pulmonary embolism. 2. Bronchoscopy is used to evaluate tissue within the bronchial tubes. 4. A biopsy
would be contraindicated for a patient with a diagnosis related to blood clotting.

73 A child is brought to the emergency department by his mother after he lacerated his scalp in a fall.
She is very anxious about the amount of blood that is oozing from his head. The nurse explains to the
mother that bright red blood signals that this type of bleeding is:

1. Capillary. 2. Venous. 3. Arterial. 4. Afferent.

1. Capillary blood is bright red and oozes from a wound. 2. Venous bleeding is dark in color and flows
steadily. 3. Arterial blood is bright, but it spurts. 4. Afferent describes the direction that blood is flowing
in a vessel and can be used in reference to both arteries and veins, depending on their function.

74 In a conscious infant who loses consciousness, removing a foreign body from his airway should entail:

1. Shining an examination light into the mouth to visualize the object. 2. Vigorous finger sweeps to
palpate the object. 3. Removing the object only if it is visible. 4. Applying pressure on the cricoid
cartilage to expel the object.

3. The object should be removed only if it is clearly visible and can be grasped. 1. If the object is not
readily visible, no further efforts toward examination should be made because they will delay the
rescue. 2. Finger sweeps may force the object further into the airway. 4. Locating this anatomical
landmark on the neck of an infant would be difficult, and pressure on the neck or throat area would not
facilitate removal of a foreign body in the airway.

75 In triage of a disaster event, the civilian priority is care to the victims who:
1. Are most likely to survive. 2. Have children. 3. Have the least serious wounds and can be cared for
quickly. 4. Have the most serious, life-threatening injuries.

4. The civilian priority is for the care of patients whose lives are threatened. 1. The military priority is
for those who are most likely to survive. 2. Triage is done on the basis of risk to the patient, not to
survivors or family members. 3. This choice places people already at the greatest risk in further danger
of morbidity or mortality.

76 A 3-year-old child is brought to the emergency department after a motor vehicle accident. She has a
scalp laceration that is oozing blood; pain, discoloration, and swelling of her right lower arm; and right-
sided chest pain that increases on inspiration. The nursing actions with the highest priority are:

1. Administration of analgesic medication and tetanus toxoid. 2. Immobilization and elevation of the
right arm. 3. Assessment and maintenance of airway and adequate ventilation. 4. Application of ice
packs and pressure dressings to the scalp laceration.

3. Assessing and maintaining an airway and adequate ventilation are always the highest priorities. 1.
Administering medication is not as high a priority as maintaining an airway and ventilation. 2. A
fractured arm is a lesser priority compared with airway and ventilation. 4. Given that the scalp laceration
is oozing and not bleeding freely, it is not as great a priority compared with airway and ventilation.

77 A victim of domestic violence is brought to the emergency department by the police. She is
complaining of abdominal pain where she was struck forcibly with a baseball bat. On examination, she is
found to have guarding, diminished bowel sounds, and bruising on the abdomen. The nurse explains to
her that she may undergo which procedure?

1. Peritoneal lavage 2. Gastric lavage 3. Barium enema 4. Abdominal MRI

1. Diagnostic peritoneal lavage helps determine whether an intraabdominal injury exists and whether
surgery is required. 2. Gastric lavage is indicated in cases of ingestion of drugs or poisons. 3. As a
radiological examination of the lower intestine, the barium enema would not be the best way to obtain
information about possible internal injuries to the abdomen. 4. Although abdominal MRI would show
information about abdominal trauma, peritoneal lavage provides a method of intervention and
treatment as well, and it is much more economical.

78 A nurse arrives at the scene of a motor vehicle accident and quickly assesses the injured parties. The
priority care should be given to the:

1. Driver with a head laceration and severe knee pain. 2. Passenger with a bleeding nose who is
complaining of a headache. 3. Driver with a hoarse voice who is coughing blood. 4. Passenger with an
arm laceration who is crying hysterically.

3. The victim who is hoarse and coughing blood may have serious injuries to the throat that can
compromise the airway. 1. This victim has no apparent injuries that would compromise his or her
airway or circulation. 2. This victim should be leaned forward to prevent aspiration of blood but is not a
treatment priority. 4. This victim may have the least serious injury and is not the greatest priority.
79 During afternoon rounds the nurse discovers a 60-year-old patient with diabetes unconscious on the
floor near his bed. The nurse’s immediate action should be to:

1. Call the physician and prepare intravenous glucose. 2. Assess for breathing and circulation. 3. Check
the blood glucose and administer regular insulin. 4. Check for medical alert identification and call the
family.

2. The priority for any unconscious victim is to assess for breathing and circulation. 1. This assumes
that the patient is unconscious because of insulin shock. 3. This assumes that the patient is unconscious
because of hypoglycemia. 4. Notification of the family is a lower priority than patient interventions.

81 A 46-year-old man who was shoveling his driveway all morning is brought to the urgent care clinic by
his wife. The nurse notes that his toes are numb, discolored, and blistered. She explains to them that
these signs indicate:

1. Immersion foot. 2. Chilblain. 3. Frostbite. 4. Hypothermia.

3. Deep frostbite is characterized by hyperemic skin, numbness, blister formation, and edema. 1.
Immersion foot gives the extremity a shriveled appearance. 2. Burning, itching, and ulcerations
characterize chilblain. 4. Hypothermia is a generalized condition characterized by tachypnea,
tachycardia, and confusion. Frostbite is a localized condition.

82 What would care of the patient with a gunshot injury include? Select all that apply.

______1. Documenting the exact location of the wound ______2. Recording the number of wounds
______3. Placing the patient’s clothing in a paper or plastic bag and securing it for the police ______4.
Placing paper bags over hands to protect evidence of gunshot residue

__X__ 1. Documenting exact location of wounds is an appropriate intervention. __X__ 2. Recording the
number of wounds is an appropriate intervention. __X__ 3. Placing the clothing of the victim in a paper
bag for authorities is an appropriate intervention. _____ 4. Specific testing would reveal evidence of
gunshot residue; no special precautions are necessary.

83 An ambulatory patient enters the emergency department after exposure to a toxic substance. Place
the steps in the process of decontamination of the victim in order of priority from first step to last.

1. Place the patient’s valuables in a plastic zipper bag, label, and give to the patient. 2. Brush off visible
dry material from the patient or patient’s clothing. 3. Remove patient’s clothing. 4. Use a paper towel or
diaper to absorb blistering agents.

Correct order: 2431. 2. Brush off particles. 4. Use paper towel to absorb blistering agents. 3. Remove
clothing. 1. Place valuables in plastic bag. The nurse’s interventions in order of priority are to brush off
particles, use a paper towel to absorb blistering agents, remove clothing, and place valuables in plastic
bag.

84 A 49-year-old patient was brought to the emergency department by a friend. The patient was
complaining of cough, dyspnea, and aching chest pain. On auscultation the nurse notes a grating sound
on inspiration that is similar to pieces of leather rubbing together. The nurse recognizes this to be an
abnormal breath sound called:

1. Egophony. 2. Bronchovesicular. 3. Pleural friction rub. 4. Crackles.

3. A pleural friction rub is the sound heard when there is a collection of fluid in the pleura. 1.
Egophony is a symptom of pleural effusion, but it is not a grating sound. 2. Bronchovesicular sounds are
normal breath sounds. 4. Crackles have a popping sound.

86 A 72-year-old biker is brought into the urgent care center after complaining of headache pain and
vertigo. The nurse assesses the patient and finds unequal hand grips, with the left hand weaker than the
right and an unequal smile. The patient cannot puff out his cheeks bilaterally. The nurse knows that
these may be symptoms of:

1. Hyperglycemic reaction. 2. Pulmonary emboli. 3. Cerebrovascular accident. 4. Rheumatoid arthritis.

3. Unequal hand grips and smile are indicators of cerebrovascular accident. 1. Unequal hand grips and
smile are not indicators of hyperglycemic reaction. 2. Chest pain and dyspnea indicate pulmonary
emboli. 4. Joint pain and deformity are more commonly seen in rheumatoid arthritis.

87 A 22-month-old is brought to the emergency department with multiple white pustules covering the
chest and face. The physician has made the diagnosis of smallpox. The nurse would follow which
bioterrorism infection control practices for patient management? Select all that apply.

______1. Contact Precautions ______2. Semiprivate room with the door opened ______3. Airborne
precautions with an N95 mask ______4. Discharge of the toddler after 72 hours of antibiotic therapy
______5. No restrictions relating to patient transport

__X__ 1. Contact Precautions are an appropriate intervention. _____ 2. A private room with door closed
is appropriate. __X__ 3. Airborne Precautions with an N95 mask are an appropriate intervention. _____
4. Discharge would not occur after 72 hours of antibiotic therapy. _____ 5. Patient must wear mask and
be gowned or wrapped in a sheet so rash is fully covered when he or she is outside of isolation room.

89 Select all clinical manifestations of intestinal trauma.

______1. Rebound tenderness ______2. Hypoactive bowel sounds ______3. Joint pain ______4.
Abdominal guarding ______5. Hemiparesis
__X__ 1. Rebound tenderness is a typical symptom of intestinal trauma that a patient would exhibit,
depending on the trauma. __X__ 2. Hypoactive bowel sounds are a typical symptom of intestinal trauma
that a patient would exhibit, depending on the trauma. _____ 3. Joint pain is commonly noted in
arthritis such as osteoarthritis and rheumatoid arthritis. __X__ 4. Abdominal guarding is a typical sign of
intestinal trauma that a patient would exhibit, depending on the trauma. _____ 5. Hemiparesis is a
muscular weakness of one half (one side) of the body.

90 Rabies is transmitted to humans through a bite from an infected animal. The causative agent is a:

1. Bacterium. 2. Fungus. 3. Virus. 4. Helminth.

91 Select all that apply to anaphylactic shock.

______1. It is a severe allergic reaction. ______2. It is a profound antibody response to an antigen.


______3. It causes increased capillary permeability and angioedema. ______4. It is caused by an
overwhelming bacterial infection. ______5. It causes bloody stools.

3. Rabies is a viral infection. 1. Bacteria are small unicellular microorganisms that vary in shape; they
can be spherical (cocci), rod-shaped (bacilli), spiral (spirochete), or comma-shaped (vibrios). Bacteria do
not cause rabies. 2. Fungus does not cause rabies. Fungi may be saprophytes or parasites. Yeasts
reproduce by budding, and molds reproduce by spore formation. 4. Helminths are worms of the
pathogenic parasite division such as tapeworms and roundworms.

93 Select all that apply to cardiogenic shock.

______1. It often leads to death within 24 hours. ______2. It decreases cardiac output. ______3. It
lowers blood pressure. ______4. It results in decreased oxygen to the tissues. ______5. It causes an
increase in urinary output.

_____ 1. Cardiogenic shock can occur within 24 hours after an acute myocardial infarction in patients
with severely impaired pumping action of the heart. __X__ 2. Cardiogenic shock decreases cardiac
output. __X__ 3. Cardiogenic shock lowers blood pressure. __X__ 4. Cardiogenic shock decreases blood
flow to tissues, resulting in decreased oxygen to the tissues. _____ 5. Urinary output is decreased in
cardiogenic shock.

97 A bite from a brown recluse spider produces the following local reactions. Select all that apply.

______1. A blue ring around the bite site ______2. Edema and blistering 2 to 8 hours after the bite
______3. A stinging sensation that may go unnoticed ______4. Muscle spasms ______5. Seizures
__X__ 1. A blue ring around the bite site is symptomatic of a brown recluse spider bite. __X__ 2. Edema
and blistering are symptomatic of a brown recluse spider bite. __X__ 3. A stinging sensation that goes
unnoticed is symptomatic of a brown recluse spider bite. _____ 4. Muscle spasms are typically noted
from the bite of a black widow spider. _____ 5. Seizures are typically noted from the bite of a black
widow spider.

98 The nurse knows that the hepatitis C virus attacks the liver and can cause cancer and death.
Transmission of hepatitis C can occur through which of the following? Select all that apply.

______1. Needle sharing ______2. Kissing and hugging ______3. Body piercing with a contaminated
needle ______4. Drinking glass ______5. Uncooked meat

__X__ 1. Needle sharing is a common mode of transmission of hepatitis C. _____ 2. Kissing and hugging
do not transmit hepatitis C. __X__ 3. Body piercing with a contaminated needle is a mode of
transmission of hepatitis C. _____ 4. Hepatitis B, C, D, and G are transmitted by a blood or sexual
transmission route. _____ 5. Hepatitis B, C, D, and G are transmitted by a blood or sexual transmission
route.

99 A 90-year-old patient who resides in a nursing home fall and sustains a left hip fracture of the
acetabulum, greater trochanter, and femoral head. The nurse knows that the symptoms of hip fracture
may include which of the following? Select all that apply.

______1. Trousseau sign ______2. Severe pain with movement of the limb ______3. External rotation of
the limb ______4. Fever and night sweats ______5. Lengthening of the affected limb

_____ 1. Trousseau sign is an indicator of brachial blood flow. __X__ 2. Severe pain with movement is a
symptom of fractured hip. __X__ 3. External rotation of the affected limb is a symptom of a fractured
hip. _____ 4. Fever is noted in the early stages of tuberculosis, and night sweats occur as the disease
progresses. _____ 5. Shortening, not lengthening, of the affected limb is seen in a fractured hip. 100.
Knowledge, assessment, physiologica

Reference: Eyles, Mary O - Mosby's comprehensive review of practical nursing for the NCLEX-PN
examination-Elsevier Health Sciences_Elsevier Mosby (2014)

You might also like